<div dir="ltr"><div>Dear Chip,<br><br></div>I look forward to the time when I have more available to give your views the full effort and thought they deserve. ( I have not even had time to read <span style="color:black">Christian’s work yet). </span>Some quick comments below.<br><div><div><div class="gmail_extra"><br><div class="gmail_quote">On Sat, Apr 4, 2015 at 2:41 AM, Chip Akins <span dir="ltr"><<a href="mailto:chipakins@gmail.com" target="_blank">chipakins@gmail.com</a>></span> wrote:<br><blockquote class="gmail_quote" style="margin:0px 0px 0px 0.8ex;border-left:1px solid rgb(204,204,204);padding-left:1ex"><div link="blue" vlink="purple" lang="EN-US"><div><p class="MsoNormal"><span style="color:black">Hi Andrew<u></u><u></u></span></p><p class="MsoNormal"><span style="color:black"><u></u> <u></u></span></p><p class="MsoNormal"><span style="color:black">Thank you for those references.<u></u><u></u></span></p><p class="MsoNormal"><span style="color:black"><u></u> <u></u></span></p><p class="MsoNormal"><span style="color:black">For a few reasons I had temporarily ruled out the notion that photon spin can be in any axis other than around the longitudinal.  One reason is that if spin is parallel to the plane of the longitudinal axis it requires the field velocity for half the cycle to be twice the speed c and for the other half cycle to reach zero.  Neither of which seem reasonable.</span></p></div></div></blockquote><div><br></div><div>I think that this can be a 'given'. However, in the context of standing waves the net energy transport would not exceed the speed of light and local 'motion' might be able to exceed c. Whether it can produce photon spin with an axis along the velocity vector and still be linear is the question.<br></div><blockquote class="gmail_quote" style="margin:0px 0px 0px 0.8ex;border-left:1px solid rgb(204,204,204);padding-left:1ex"><div link="blue" vlink="purple" lang="EN-US"><div><p class="MsoNormal"><span style="color:black"><u></u><u></u></span></p><p class="MsoNormal"><span style="color:black"><u></u> <u></u></span></p><p class="MsoNormal"><span style="color:black">Due to the wide use of down-converting crystals to create a pair of “entangled” photons, at half the frequency (energy) of the original incident photon, for photon polarization “entanglement” experiments, I am similarly temporarily setting aside the notion that polarization is a multiple photon effect. For if it required more than one photon to constructively interfere to pass through a polarizer, those experiments would be much less informative than they are currently believed to be by most physicists.  The nature of those experiments indicates the belief that photon polarization is a single photon phenomena.</span></p></div></div></blockquote><div><br></div><div>Some interferometer experiments seem to indicate that single photons can actually split and go both ways - but only if they can recombine at some later point.<br></div><blockquote class="gmail_quote" style="margin:0px 0px 0px 0.8ex;border-left:1px solid rgb(204,204,204);padding-left:1ex"><div link="blue" vlink="purple" lang="EN-US"><div><p class="MsoNormal"><span style="color:black"><u></u><u></u></span></p><p class="MsoNormal"><span style="color:black"><u></u> <u></u></span></p><p class="MsoNormal"><span style="color:black">One popular interpretation of experiment uses the quantum notion of superposition of spin states to explain polarization.  But this seems to be an unphysical explanation to me, so I am looking at possible alternates, with a more causal and classical description.</span></p></div></div></blockquote><div><br></div><div>me too! <br></div><blockquote class="gmail_quote" style="margin:0px 0px 0px 0.8ex;border-left:1px solid rgb(204,204,204);padding-left:1ex"><div link="blue" vlink="purple" lang="EN-US"><div><p class="MsoNormal"><span style="color:black"><u></u><u></u></span></p><p class="MsoNormal"><span style="color:black"><u></u> <u></u></span></p><p class="MsoNormal" style="text-align:justify"><span style="color:black">One of those alternates is described as follows… It may be that a photon can have any spin value up to the limit +/- ħ. If I remember correctly Chandra and John W. have commented on this and may have more information. If this is the case then a single photon may be able to consist of more than one set of fields. The frequency and confinement (quantization) we have discussed, will still work in that scenario, because</span><span style="font-family:"Calibri",sans-serif;color:black"> </span><span style="color:black">it is the energy density in that volume of space which causes the total EM forces which also would produce quantization, (frequency, and confinement).  It seems that a single photon is always monochromatic, so the frequency and wavelength would be the same for each set of fields (waves).  However if one set of fields is “pulling left” and the other set is “pulling right” then the composite set of fields displays a lower spin angular momentum value, and even a zero spin angular momentum value, for a plane polarized photon with balanced and equal sets of fields spinning opposite directions. Of course we know, and can demonstrate easily, that EM waves can pass through one another and coexist, so it is not difficult to imagine these photon fields behaving in such a manner.  Quantum physics posits the superposition of two spin states with an entangled wave function collapse at the measurement of one of the entangled particles. Joy Christian’s work shows how two non-commuting spin operators, as local variables, satisfy the same Bell’s inequalities. Both explanations (quantum and non-commuting local variable) can be realized by a classical, causal based physical topology, with two sets of fields, each exhibiting spin, but in opposite directions. If this scenario is accurate it would imply that plane polarized photons would behave slightly differently in some diffraction experiments, than circularly polarized photons. Such an effect has already been shown in experiment, using the polarization dependence of Fraunhofer diffraction and a metallic grating. <u></u><u></u></span></p><p class="MsoNormal" style="text-align:justify"><span style="color:black"><u></u> <u></u></span></p><p class="MsoNormal" style="text-align:justify"><span style="color:black">Thoughts?</span></p></div></div></blockquote><div><br></div><div>Since photons must be composed of (decomposed into?) many waves of different frequencies, then  see no reason not to mix spin components. Until you suggested that, I would have been very resistant to the idea of non-unit spin. I am reluctant to give physical identity to the individual components of the Fourier decomposition. Nevertheless, I can easily believe that they are to be represented in the interactions with matter (or perhaps with other  photons. My feeling that photons are solitons/resonances discourages, but does not eliminate, the mixture of counter-rotating components.<br><br></div><div>Do you have any references for <span style="color:black">the polarization dependence of Fraunhofer diffraction and a metallic grating?</span><span style="color:black"><br><br></span></div><div><span style="color:black">Re your notes to John M and Richard:  </span>You have raised the question of "t<span style="color:black">opological origin of charge." I believe that this is a very important concept that has been ignored. Bob Hudgins' thoughts, developed in the analysis of standing waves, have led to a paper on the topic to be presented in San Diego. If Bob is not feeling up to presenting up to presenting a second (or third) paper at the conference and our other coauthor, Ralph Penland, is not able to attend, we might ask you or someone else in the group to present it for us. [My wife is expecting twins in September and that has screwed up our plans to attend the conference and then fly back to India, which were made when we were expecting only a single baby.]<br><br></span></div><div><span style="color:black">Thanks again for your thoughts and contributions.<br><br></span></div><div><span style="color:black">Andrew<br></span></div><div><span style="color:black">_______________________________<br></span></div><blockquote class="gmail_quote" style="margin:0px 0px 0px 0.8ex;border-left:1px solid rgb(204,204,204);padding-left:1ex"><div link="blue" vlink="purple" lang="EN-US"><div><p class="MsoNormal"><b><span style="font-size:11pt;font-family:"Calibri",sans-serif">From:</span></b><span style="font-size:11pt;font-family:"Calibri",sans-serif"> General [mailto:<a href="mailto:general-bounces%2Bchipakins" target="_blank">general-bounces+chipakins</a>=<a href="mailto:gmail.com@lists.natureoflightandparticles.org" target="_blank">gmail.com@lists.natureoflightandparticles.org</a>] <b>On Behalf Of </b>Andrew Meulenberg<br><b>Sent:</b> Friday, April 03, 2015 12:27 PM<br><b>To:</b> Nature of Light and Particles - General Discussion; Andrew Meulenberg; Jean-Luc Pierre P.<span class=""><br><b>Subject:</b> Re: [General] Nature of Light and Particles - Request<u></u><u></u></span></span></p><p class="MsoNormal"><u></u> <u></u></p><div><div><div><div><p class="MsoNormal" style="margin-bottom:12pt">Dear Chip,<u></u><u></u></p></div><div><div class="h5"><p class="MsoNormal" style="margin-bottom:12pt">Since I have been having this discussion with someone (Jean-Luc Paillet) in a different context, I thought that I would take the time to try and find a paper that contained a statement that I had interpreted to mean that a linear-polarized photon still had a spin of 1. <br><br>I found what I think may be what I had seen (attached). However, now that I look more closely, I am not sure that it is referring to a photon or a collection. Perhaps someone more mathematically sophisticated can look at sections 6.7 (for circular-polarization) and 6.8 (for linear-polarization) of the attached and let me know if it can refer to single photons as well as collections. "We recover the classical result derived in Section 6.7: the spin is in the direction of propagation of the wave."<u></u><u></u></p></div></div></div><div><div class="h5"><p class="MsoNormal" style="margin-bottom:12pt">Jean-Luc referred to the 3rd from last paragraph of <a href="http://mathpages.com/rr/s9-04/9-04.htm" target="_blank">http://mathpages.com/rr/s9-04/9-04.htm</a> , which states that linear-polarized light is only balanced circular-polarized light. However, it further states that individual photons will register as +/- hbar. Thus, it is a superposition of 2 states, rather than a 3rd state. If this is the case, does the E = n h nu relation come into play? If so, then I assume that spectrometers could respond differently to  linear- and circular-polarized light of the same energy (with n = 2 and 1 respectively). On the other hand, since w = w1+/- w2, a spectrometer might see only the sum of the two coherent photons (a thermally stable BEC?). It is an interesting problem that I see no convincing solution to.<u></u><u></u></p></div></div></div><div><div class="h5"><p class="MsoNormal">Andrew<u></u><u></u></p><div><div><div><div><div><p class="MsoNormal" style="margin-bottom:12pt">______________________--<u></u><u></u></p><div><p class="MsoNormal">On Fri, Apr 3, 2015 at 7:26 PM, Chip Akins <<a href="mailto:chipakins@gmail.com" target="_blank">chipakins@gmail.com</a>> wrote:<u></u><u></u></p><blockquote style="border-width:medium medium medium 1pt;border-style:none none none solid;border-color:-moz-use-text-color -moz-use-text-color -moz-use-text-color rgb(204,204,204);padding:0in 0in 0in 6pt;margin-left:4.8pt;margin-right:0in"><div><div><p class="MsoNormal"><span style="color:black">Hi John W</span><u></u><u></u></p><p class="MsoNormal"><span style="color:black"> </span><u></u><u></u></p><p class="MsoNormal"><span style="color:black">The intent of this line of discussion is to probe more deeply into the structure of the photon and to address polarization entanglement experiments.</span><u></u><u></u></p><p class="MsoNormal"><span style="color:black"> </span><u></u><u></u></p><p class="MsoNormal"><span style="color:black">A thought and some questions for you John.</span><u></u><u></u></p><p class="MsoNormal"><span style="color:black"> </span><u></u><u></u></p><p class="MsoNormal"><span style="color:black">First some background.  As I understand it Quantum physics posits a superposition of spin states as a cause for planar polarization. In order to reach a more causal explanation, can we then envision two fields within the photon, spinning opposite directions, and constructively interfering only in a plane, which is dependent on their spin phase? </span><u></u><u></u></p></div></div></blockquote><div><p class="MsoNormal"> <u></u><u></u></p></div><blockquote style="border-width:medium medium medium 1pt;border-style:none none none solid;border-color:-moz-use-text-color -moz-use-text-color -moz-use-text-color rgb(204,204,204);padding:0in 0in 0in 6pt;margin-left:4.8pt;margin-right:0in"><div><div><p class="MsoNormal"><span style="color:black">Are you familiar with Joy Christian’s work?  He writes that two non-commuting rotations (spin operators) as local variables, exactly duplicate the predictions of Quantum mechanics and satisfy Bell’s inequalities in precisely the same way. I have checked some of the math and so far it seems to be quite accurate. In both of these approaches, two oppositely rotating fields would apparently satisfy these physical aspects of the theories… ???</span><u></u><u></u></p><p class="MsoNormal"><span style="color:black"> </span><u></u><u></u></p><p class="MsoNormal"><span style="color:black">Christian uses a Clifford algebra to illustrate his theory.  Have you had the chance to compare that with the work you are doing using Clifford algebra to in your new theory of light and matter?  Specifically have you had any opportunity to check to see if two opposite, (non-commuting local) spins caused by your framework would also satisfy Bell’s inequalities? Or CHSH inequalities?</span><u></u><u></u></p><p class="MsoNormal"><span style="color:black"> </span><u></u><u></u></p><p class="MsoNormal"><span style="color:black">Of course you can see the underlying reasons for these questions.  One underlying reason is to discover if two equal and oppositely spinning fields, confined within the photon, can explain polarization.  In both, quantum physics, and Christian’s theories, it seems that two opposite spins are required, hinting that we would need those two opposite physical spins to be possible in a physical model of the photon.</span><u></u><u></u></p><p class="MsoNormal"><span style="color:black"> </span><u></u><u></u></p><p class="MsoNormal"><span style="color:black">The other underlying reason is to discover if non-commuting (rotation) local variables can potentially be the cause for the appearance of entanglement.</span><u></u><u></u></p><p class="MsoNormal"><span style="color:black"> </span><u></u><u></u></p><p class="MsoNormal"><span style="color:black">Thoughts?</span><u></u><u></u></p><p class="MsoNormal"><span style="color:black"> </span><u></u><u></u></p><p class="MsoNormal"><span style="color:black">Chip</span><br></p></div></div></blockquote></div></div></div></div></div></div></div></div></div></div></div></blockquote></div><br></div></div></div></div>